Difference between revisions of "2018 AIME I Problems/Problem 13"

(Solution (Add and multiply only))
(Solution (Add and multiply only))
Line 40: Line 40:
 
https://www.youtube.com/watch?v=sT-wxV2rYqs
 
https://www.youtube.com/watch?v=sT-wxV2rYqs
  
==Solution (Add and multiply only)==
+
==Solution (Geometry only)==
 
[[File:2018 AIME I 13.png|400px|right]]
 
[[File:2018 AIME I 13.png|400px|right]]
 
Let <math>BC = a, s</math> be semiperimeter of <math>\triangle ABC, s = 48, h</math> be the height of <math>\triangle ABC</math> dropped from <math>A.</math>
 
Let <math>BC = a, s</math> be semiperimeter of <math>\triangle ABC, s = 48, h</math> be the height of <math>\triangle ABC</math> dropped from <math>A.</math>
Line 69: Line 69:
 
<cmath>2[ABX] = r_1 (c + t + x_1) = h x_1 \implies \frac{h}{r_1} = 1 + u,</cmath>
 
<cmath>2[ABX] = r_1 (c + t + x_1) = h x_1 \implies \frac{h}{r_1} = 1 + u,</cmath>
 
<cmath>2[ACX] = r_2 (b + t + x_2) = h x_2 \implies \frac{h}{r_2} = 1 + v,</cmath>
 
<cmath>2[ACX] = r_2 (b + t + x_2) = h x_2 \implies \frac{h}{r_2} = 1 + v,</cmath>
 +
<cmath>\frac{h}{r_2} +  \frac{h}{r_1}- 2 = \frac{h}{r_2}  \frac{h}{r_1} \frac{r}{h}  \iff (u+v)(a+b+c)=(u+1)(v+1)a \iff (u+v)(b+c)=(uv+1)a,</cmath>
 +
<cmath>c^2 x_2 + b^2 x_1 = t^2 x_1 + t^2 x_2 + x_1^2 x_2 + x_1 x_2^2,</cmath>
 +
<cmath>(b^2 -t^2 -x_2^2)x_1 = (c^2 – t^2 -x_1^2)x_2 = 0,</cmath>
 +
We use Cosine Law for <math>\triangle ABX</math> and <math>\triangle ACX</math> and get
 +
<cmath>2 t x_1 x_2 \cos \angle AXB +  2 t x_1 x_2 \cos \angle AXC = 0.</cmath>
 +
The claim has been proven.
 +
 +
~vvsss, www.deoma-cmd.ru
  
 
==See Also==
 
==See Also==

Revision as of 15:00, 15 June 2022

Problem

Let $\triangle ABC$ have side lengths $AB=30$, $BC=32$, and $AC=34$. Point $X$ lies in the interior of $\overline{BC}$, and points $I_1$ and $I_2$ are the incenters of $\triangle ABX$ and $\triangle ACX$, respectively. Find the minimum possible area of $\triangle AI_1I_2$ as $X$ varies along $\overline{BC}$.

Solution 1 (Official MAA)

First note that \[\angle I_1AI_2 = \angle I_1AX + \angle XAI_2 = \frac{\angle BAX}2 + \frac{\angle CAX}2 = \frac{\angle A}2\] is a constant not depending on $X$, so by $[AI_1I_2] = \tfrac12(AI_1)(AI_2)\sin\angle I_1AI_2$ it suffices to minimize $(AI_1)(AI_2)$. Let $a = BC$, $b = AC$, $c = AB$, and $\alpha = \angle AXB$. Remark that \[\angle AI_1B = 180^\circ - (\angle I_1AB + \angle I_1BA) = 180^\circ - \tfrac12(180^\circ - \alpha) = 90^\circ + \tfrac\alpha 2.\] Applying the Law of Sines to $\triangle ABI_1$ gives \[\frac{AI_1}{AB} = \frac{\sin\angle ABI_1}{\sin\angle AI_1B}\qquad\Rightarrow\qquad AI_1 = \frac{c\sin\frac B2}{\cos\frac\alpha 2}.\] Analogously one can derive $AI_2 = \tfrac{b\sin\frac C2}{\sin\frac\alpha 2}$, and so \[[AI_1I_2] = \frac{bc\sin\frac A2 \sin\frac B2\sin\frac C2}{2\cos\frac\alpha 2\sin\frac\alpha 2} = \frac{bc\sin\frac A2 \sin\frac B2\sin\frac C2}{\sin\alpha}\geq bc\sin\frac A2 \sin\frac B2\sin\frac C2,\] with equality when $\alpha = 90^\circ$, that is, when $X$ is the foot of the perpendicular from $A$ to $\overline{BC}$. In this case the desired area is $bc\sin\tfrac A2\sin\tfrac B2\sin\tfrac C2$. To make this feasible to compute, note that \[\sin\frac A2=\sqrt{\frac{1-\cos A}2}=\sqrt{\frac{1-\frac{b^2+c^2-a^2}{2bc}}2} = \sqrt{\dfrac{(a-b+c)(a+b-c)}{4bc}}.\] Applying similar logic to $\sin \tfrac B2$ and $\sin\tfrac C2$ and simplifying yields a final answer of \begin{align*}bc\sin\frac A2\sin\frac B2\sin\frac C2&=bc\cdot\dfrac{(a-b+c)(b-c+a)(c-a+b)}{8abc}\\&=\dfrac{(30-32+34)(32-34+30)(34-30+32)}{8\cdot 32}=\boxed{126}.\end{align*}


Solution 2 (A lengthier, but less trigonometric approach)

First, instead of using angles to find $[AI_1I_2]$, let's try to find the area of other, simpler figures, and subtract that from $[ABC]$. However, to do this, we need to be able to figure out the length of the inradii, and so, we need to find $AX$.


To minimize $[AI_1I_2]$, intuitively, we should try to minimize the length of $AX$, since, after using the $rs=A$ formula for the area of a triangle, we'll be able to minimize the inradii lengths, and thus, eventually minimize the area of $[AI_1I_2]$. (Proof needed here).


We need to minimize $AX$. Let $AX=d$, $BX=s$, and $CX=32-s$. After an application of Stewart's Theorem, we will get that \[d=\sqrt{s^2-24s+900}\] To minimize this quadratic, $s=12$ whereby we conclude that $d=6\sqrt{21}$.


From here, draw perpendiculars down from $I_1$ and $I_2$ to $AB$ and $AC$ respectively, and label the foot of these perpendiculars $D$ and $E$ respectively. After, draw the inradii from $I_1$ to $BX$, and from $I_2$ to $CX$, and draw in $I_1I_2$.

Label the foot of the inradii to $BX$ and $CX$, $F$ and $G$, respectively. From here, we see that to find $[AI_1I_2]$, we need to find $[ABC]$, and subtract off the sum of $[DBCEI_2I_1], [ADI_1],$ and $[AEI_2]$.


$[DBCEI_2I_1]$ can be found by finding the area of two quadrilaterals $[DBFI_1]+[ECGI_2]$ as well as the area of a trapezoid $[FGI_2I_1]$. If we let the inradius of $ABX$ be $r_1$ and if we let the inradius of $ACX$ be $r_2$, we'll find, after an application of basic geometry and careful calculations on paper, that $[DBCEI_2I_1]=13r_1+19r_2$.


The area of two triangles can be found in a similar fashion, however, we must use $XYZ$ substitution to solve for $AD$ as well as $AE$. After doing this, we'll get a similar sum in terms of $r_1$ and $r_2$ for the area of those two triangles which is equal to \[\frac{(9+3\sqrt{21})(r_1)}{2} + \frac{(7+3\sqrt{21})(r_2)}{2}.\]

Now we're set. Summing up the area of the Hexagon and the two triangles and simplifying, we get that the formula for $[AI_1I_2]$ is just \[[ABC]-\left(\frac{(35+3\sqrt{21})(r_1)}{2}+\frac{(45+3r_2\sqrt{21})(r_2)}{2}\right).\]

Using Heron's formula, $[ABC]=96\sqrt{21}$. Solving for $r_1$ and $r_2$ using Heron's in $ABX$ and $ACX$, we get that $r_1=3\sqrt{21}-9$ and $r_2=3\sqrt{21}-7$. From here, we just have to plug into our above equation and solve.

Doing so gets us that the minimum area of $AI_1I_2=\boxed{126}.$

-Azeem H.(Mathislife52) ~edited by phoenixfire

Video Solution by Osman Nal

https://www.youtube.com/watch?v=sT-wxV2rYqs

Solution (Geometry only)

2018 AIME I 13.png

Let $BC = a, s$ be semiperimeter of $\triangle ABC, s = 48, h$ be the height of $\triangle ABC$ dropped from $A.$

Let $r, r_1, r_2$ be inradius of the $\triangle ABC, \triangle ABX,$ and $\triangle ACX,$ respectively.

Using the Lemma (below), we get the area \[[ AI_1 I_2] = \frac{AX \cdot r }{2} \ge \frac { hr}{2} =\frac{ r^2 s}{a},\] \[[ AI_1 I_2]=  \frac{(s-a)(s-b)(s-c)}{a} =   \frac{18 \cdot 16 \cdot 14}{32} = 9 \cdot 14 = \boxed {126}.\] Lemma

\[[AI_1 I_2] = \frac{AX \cdot r }{2}\]

Proof \[\angle AXI_1 = \angle BXI_1, \angle AXI_2 = \angle CXI_2, \angle BXC = 180^\circ \implies \angle I_1 X I_2 = 90^\circ.\] WLOG $\hspace{70mm} \sin \angle AXB = \frac {h}{AX}.$ \[[AI_1 X] =\frac{ AX r_1}{2}, \hspace{20mm} [AI_2 X] =\frac{ AX r_2}{2},\hspace{20mm}[XI_1 I_2] =\frac{XI_1 \cdot XI_2}{2},\] \[XI_1  \cdot XI_2 = \frac{r_1}{\sin \angle AXI_1} \cdot \frac{r_2}{\sin \angle AXI_2} =  \frac{2 r_1 r_2}{\sin \angle AXB} =  \frac{2 AX r_1 r_2 }{h}.\] $[AI_1 I_2] = [AI_1 X] + [AI_2 X] - [XI_1 I_2] = AX (r_1 + r_2 -  \frac{2 r_1 r_2 }{h}) =  \frac{AX \cdot r }{2}$ if and only if

Claim \[r_1 + r_2 -  \frac{2 r_1 r_2 }{h} = r.\] Proof

Let $AX = t, AC = b, AB = c, x_1 = BX, x_2 = CX, \frac{c+t}{x_1} = u, \frac{b+t}{x_2} = v.$ \[r_1 + r_2 -  \frac{2 r_1 r_2 }{h} = r  \iff  \frac{h}{r_2} +  \frac{h}{r_1}- 2 = \frac{h}{r_2}  \frac{h}{r_1} \frac{r}{h}.\]

\[2[ABX] = r_1 (c + t + x_1) = h x_1 \implies \frac{h}{r_1} = 1 + u,\] \[2[ACX] = r_2 (b + t + x_2) = h x_2 \implies \frac{h}{r_2} = 1 + v,\] \[\frac{h}{r_2} +  \frac{h}{r_1}- 2 = \frac{h}{r_2}  \frac{h}{r_1} \frac{r}{h}  \iff (u+v)(a+b+c)=(u+1)(v+1)a \iff (u+v)(b+c)=(uv+1)a,\] \[c^2 x_2 + b^2 x_1 = t^2 x_1 + t^2 x_2 + x_1^2 x_2 + x_1 x_2^2,\] \[(b^2 -t^2 -x_2^2)x_1 = (c^2 – t^2 -x_1^2)x_2 = 0,\] We use Cosine Law for $\triangle ABX$ and $\triangle ACX$ and get \[2 t x_1 x_2 \cos \angle AXB +  2 t x_1 x_2 \cos \angle AXC = 0.\] The claim has been proven.

~vvsss, www.deoma-cmd.ru

See Also

2018 AIME I (ProblemsAnswer KeyResources)
Preceded by
Problem 12
Followed by
Problem 14
1 2 3 4 5 6 7 8 9 10 11 12 13 14 15
All AIME Problems and Solutions

The problems on this page are copyrighted by the Mathematical Association of America's American Mathematics Competitions. AMC logo.png